L'Hopital's Rule: Solving Homework Statement

In summary, the student attempted to solve for x using the L'Hopital's rule but got incorrect results. The student then calculated the derivative of x^3 + x^2 + 15 incorrectly and found that the results were the same as when they used the rule.
  • #1
akaliuseheal
53
8
Homework Statement
Capture.PNG

Can I use L'Hopital's rule here. What I get as a solution is -30/-27 while in the notebook,
without using the L'Hopital's rule the answer is -(2/27)

The attempt at a solution
The derivatives i get are:
x/(x2+5)½
(3x2+2x)/3(x3+x2+15)⅓
2x-5

½ and ⅓ are there because it's easier for me to write it here on this forum like that instead of sqrt.

So..
limx→2( ( x/(x2+5)½ - (3x2+2x)/3(x3+x2+15)⅓ ) / 2x-5 )
Image:
Capture1.PNG

x = 2 ⇒ -30/-27 (1.11)

Could't find software online to verify the solution (symbolab gave no answer) so here I am.
But symbolab did gave me the same answer as I got when I entered the expression from the second image.
 

Attachments

  • Capture.PNG
    Capture.PNG
    1.4 KB · Views: 700
  • Capture1.PNG
    Capture1.PNG
    1.9 KB · Views: 352
Physics news on Phys.org
  • #2
Firstly, please use Latex to write equations. The link to the Latex help is: https://www.physicsforums.com/help/latexhelp/

Now, derivative of ##\sqrt{x^2+5}## is ##\dfrac{1}{2} \dfrac{2x}{\sqrt{x^2+5}}##.

The other derivatives are correct.

Put these in. Your answer should come.
 
  • #4
Wrichik Basu said:
Firstly, please use Latex to write equations. The link to the Latex help is: https://www.physicsforums.com/help/latexhelp/

Now, derivative of ##\sqrt{x^2+5}## is ##\dfrac{1}{2} \dfrac{2x}{\sqrt{x^2+5}}##.

The other derivatives are correct.

Put these in. Your answer should come.

If you look at what you got and then what I got, you will see that those two are the same.
 
  • #5
akaliuseheal said:
If you look at what you got and then what I got, you will see that those two are the same.

You calculated the other derivative incorrectly. See my post.
 
  • #6
  • Like
Likes Wrichik Basu and member 587159

What is L'Hopital's Rule?

L'Hopital's Rule is a mathematical theorem that allows for the evaluation of certain types of limits by using derivatives. It is commonly used in calculus to simplify complex limit problems.

When should L'Hopital's Rule be used?

L'Hopital's Rule should only be used when evaluating a limit that takes on an indeterminate form, such as 0/0 or ∞/∞. It is not necessary to use L'Hopital's Rule for all limit problems.

How is L'Hopital's Rule applied?

To apply L'Hopital's Rule, one must take the derivative of both the numerator and denominator separately and then evaluate the limit again. This process may need to be repeated multiple times until a non-indeterminate form is reached.

What are some common mistakes when using L'Hopital's Rule?

Some common mistakes when using L'Hopital's Rule include forgetting to check if the limit is indeterminate, taking the derivative incorrectly, and not simplifying the final result. It is important to double check each step to avoid errors.

Are there any limitations to L'Hopital's Rule?

Yes, there are certain limits that cannot be evaluated using L'Hopital's Rule, such as limits that involve oscillating functions or limits that approach a finite value. In these cases, other methods must be used to evaluate the limit.

Similar threads

Replies
1
Views
632
  • Calculus and Beyond Homework Help
Replies
1
Views
1K
  • Calculus and Beyond Homework Help
Replies
10
Views
1K
  • Calculus and Beyond Homework Help
Replies
25
Views
355
  • Calculus and Beyond Homework Help
Replies
23
Views
1K
Replies
4
Views
1K
  • Calculus and Beyond Homework Help
Replies
9
Views
2K
  • Calculus and Beyond Homework Help
Replies
5
Views
1K
  • Calculus and Beyond Homework Help
Replies
5
Views
992
  • Calculus and Beyond Homework Help
Replies
4
Views
1K
Back
Top